PT58.S4.Q23 - craters high pressure shocks characteristic

kenglish55309kenglish55309 Free Trial Member
edited December 2015 in Logical Reasoning 37 karma
Ok..I feel there is just no way I could have got this question. It is a most strengthen question and I feel the answer choice weakens the argument. The job is to sure up the fact that a volcanic event is the probable cause rather than meteorites. The part of B that says 'no known natural cause would likely account ...' I was thinking isn't meteorite as well as volcanoes natural causes so I immediately eliminated it. What do you guys think about this question?
http://7sage.com/lsat_explanations/lsat-58-section-4-question-23/

Comments

  • J.Y. PingJ.Y. Ping Administrator Instructor
    edited May 2013 13940 karma
    This is a curve breaker question. It's very, very difficult.

    First let's make sure we understand the argument. We spot craters in a line. We know that it could be caused by (1) volcanos or (2) meteorites or (3) a combination. The stimulus precludes (3) a combination. Then, it gives us one solitary premise, that the craters are different ages. On that premise it concludes that the cause is (1) volcano. But, why? Why not (2) meteorites? What does the craters being of different ages have to do with it?

    That right there makes this question hard. To strengthen an argument, you can't just prove the conclusion any way you want. You have to make your existing premise more relevant to your conclusion. [Review your lesson on how to do strengthening questions.]

    If you didn't understand that, you'll likely choose (A), which attempts to prove the conclusion independently. Most students choose this trap answer choice.

    You're likely misreading (B). (B) specifically calls out the craters as meteorite craters. If no known natural causes could have been responsible for (1) meteorite craters (2) of different ages (3) forming in a straight line, then what do you think explains the craters? Volcanos. Do you see how this answer choices makes our solitary premise more relevant to the conclusion?

    You should watch the video explanation of this question here: http://7sage.com/lsat_explanations/lsat-58-section-4-question-23/
    In this text explanation, I'm just telling you what the errors are. The video walks you through the thought process step by step so hopefully, you can mimic it next time and get a question like this one right.
  • kenglish55309kenglish55309 Free Trial Member
    37 karma
    This question has me frazzled. So ...the answer works by eliminating the possibility that the occurrence was as a result of meteorite thereby firming the support for volcanoes? I have to take the weekend to think about this one as well as review my lessons on strengthening. Thanks very much for the explanation.
  • J.Y. PingJ.Y. Ping Administrator Instructor
    13940 karma
    Sure. That's pretty much it. The answer works by "eliminating the possibility that the occurrence was as a result of meteorites thereby firming the support for volcanoes".
Sign In or Register to comment.